Peripheral Nervous System / Reflexes (Ch 13)

¡Supera tus tareas y exámenes ahora con Quizwiz!

Identify the muscle responsible for depressing the eye and turning it laterally.

A This muscle's action is to depress the eye because of its upward pulling action and also to turn the eye laterally.

B) Stimulation of the ophthalmic division of the trigeminal nerve (V) will cause blinking.

A bit of dust blows into and touches the cornea of the eye. Which of the following is likely to happen? A) Nothing, because there is no sensory information sent from the cornea. B) Stimulation of the ophthalmic division of the trigeminal nerve (V) will cause blinking. C) Stimulation of the optic nerve (II) will cause tears to flow from the lacrimal gland. D) Stimulation of the facial nerve (VII) will be perceived as pain.

A refractory period is ____.

A brief time when a neuron is unable to conduct an impulse

A) facial

A patient who received a blow to the side of the skull exhibits the following signs and symptoms on that side of the face: he is unable to close his eye, and the corner of his mouth droops. Which cranial nerve has been damaged? A) facial B) glossopharyngeal C) hypoglossal D) accessory

C) tendon organ

A person picks up a heavy suitcase in order to estimate its weight and reflexively drops it. Which of the following receptors has initiated this reflex? A) free nerve ending B) lamellae corpuscle C) tendon organ D) bulbous corpuscle

A) tendon reflex

A reflex that causes muscle relaxation and lengthening in response to muscle tension is called a ________. A) tendon reflex B) flexor reflex C) crossed-extensor reflex D) plantar reflex

Which of the receptor types might function as a nociceptor?

A. Nociceptors respond to potentially damaging stimuli like noxious chemicals and extremes of temperature or pH. Nociceptors are generally associated with free nerve endings of specialized neurons.

Elbow joint. (A) brachial (B) cervical (C) lumbar (D) sacral (1) common fibular (2) femoral (3) median (4) musculocutaneous (5) obturator (6) phrenic (7) radial (8) tibial (9) ulnar

(A) brachial (3) median (4) musculocutaneous (7) radial (9) ulnar

Muscles that flex the wrist and digits (2 nerves). (A) brachial (B) cervical (C) lumbar (D) sacral (1) common fibular (2) femoral (3) median (4) musculocutaneous (5) obturator (6) phrenic (7) radial (8) tibial (9) ulnar

(A) brachial (3) median (9) ulnar

You bump (lightly) into someone. (A) exteroceptor (B) interoceptor (C) proprioceptor (1) chemoreceptor (2) mechanoreceptor (3) nociceptor (4) photoreceptor (5) thermoreceptor

(A) exteroceptor (2) mechanoreceptor

The retinas of your eyes are stimulated. (A) exteroceptor (B) interoceptor (C) proprioceptor (1) chemoreceptor (2) mechanoreceptor (3) nociceptor (4) photoreceptor (5) thermoreceptor

(A) exteroceptor (4) photoreceptor

Serves the sternocleidomastoid and trapezius muscles. (A) abducens (B) accessory (C) facial (D) glossopharyngeal (E) hypoglossal (F) oculomotor (G) olfactory (H) optic (I) trigeminal (J) trochlear (K) vagus (L) vestibulocochlear

(B) accessory

The diaphragm. (A) brachial (B) cervical (C) lumbar (D) sacral (1) common fibular (2) femoral (3) median (4) musculocutaneous (5) obturator (6) phrenic (7) radial (8) tibial (9) ulnar

(B) cervical (6) phrenic

The neural machinery of the spinal cord is at the... (A) precommand level (B) projection level (C) segmental level

(C) segmental level

Proprioceptors include all of the following except..... (A) muscle spindles (B) tendon organs (C) tactile discs (D) joint kinesthetic receptors

(C) tactile discs

A reflex that causes reciprocal activation of the antagonist muscles is the... (A) crossed-extensor (B) flexor (C) tendon (D) muscle stretch

(C) tendon

Rapidly adapting deep pressure receptor. (A) bulbous corpuscles (B) tendon organ (C) muscle spindle (D) free nerve endings (E) lamellar corpuscle (F) tactile corpuscle

(E) lamellar corpuscle

Causes pupillary constriction. (A) abducens (B) accessory (C) facial (D) glossopharyngeal (E) hypoglossal (F) oculomotor (G) olfactory (H) optic (I) trigeminal (J) trochlear (K) vagus (L) vestibulocochlear

(F) oculomotor

Purely sensory (2 nerves). (A) abducens (B) accessory (C) facial (D) glossopharyngeal (E) hypoglossal (F) oculomotor (G) olfactory (H) optic (I) trigeminal (J) trochlear (K) vagus (L) vestibulocochlear

(G) olfactory (H) optic

All of the following are involved in maintaining the resting membrane potential EXCEPT:

***a) Differences in the ionic composition of interstitial and extracellular fluids ***a) Differences in the ionic composition of interstitial and extracellular fluids b) Differences in the permeability of the plasma membrane to K+ and Na+ c) Leak channels d) Sodium-potassium pump

White matter contains ____.

***a) myelinated axons & b) nonmyelinated axons

Which is NOT a function of cerebrospinal fluid?

***b) Isolates CNS neural tissue from general circulation a) Supports and nourishes brain b) Isolates CNS neural tissue from general circulation c) Cushions delicate neural structures (brain/spinal cord) d) Conducts chemical signals between parts of CNS

The autonomic nervous system consists of motor neurons that do all BUT:

***d) Provide motor fibers to the skeletal muscles a) Adjust to ensure optimal support for body activities b) Hormone secretion c) Innervate smooth, cardiac muscle, and glands ***d) Provide motor fibers to the skeletal muscles

Which of the following is NOT part of the cerebrellar anatomy?

***d) Reticular formation a) Arbor vitae b) Flocculondodular lobes c) Purknje cells d) Reticular formation

Describe the composition and function of the cell body.

--> The cell body, also known as the soma, contains the nucleus. It also contains many organelles: mitochondria, Golgi apparatus, ribosomes, and endoplasmic reticulum. One of its main functions is to produce proteins.

Which of the receptor types pictured functions exclusively as a proprioceptor?

B) Proprioceptors are sensitive to stimuli associated with body movements. The muscle spindle shown in B is responsive to muscle stretch.

Compare and contrast flexor and crossed-extensor reflexes.

--> The flexor, can also be known as the withdrawal, reflex is originally caused by a pain stimulus. This causes an automatic withdrawal of the threatened body part from the stimulus. Crossed-extensor reflexes are known as the complex spinal reflex that consists of an ipsilateral withdrawal reflex. It also consists of a contralateral extensor reflex.

Describe the anatomical relationship of the white and gray rami communicantes to the spinal nerve, and indicate the kind of fibers found in each ramus type.

--> The gray rami contain postganglionic nerve fibers from the sympathetic nervous system. The gray ramus is parallel to the white rami communicans. These convey the preganglionic sympathetic fibers into the paravertebral sympathetic ganglia.

Describe the importance of the hypothalamus in controlling the autonomic nervous system.

--> The importance of the hypothalamus in controlling the autonomic nervous system is that it is the main integration center. It coordinates the heart rate, blood pressure, and body temperature.

Postganglionic neurons are also called ganglionic neurons. Why is the latter term more accurate?

--> The latter term is more accurate as the cell body of the preganglionic neuron is in the CNS. The ganglionic neuron's cell body is actually in the autonomic ganglion, not post to the ganglion. But, its axon does lie distal so the term postganglionic axon is correct.

What is a benefit of a nerve plexus?

...

List the 6 types of Neuroglia and their functions. Make sure to mention whether they are part of the CNS or PNS.

1) astrocytes (CNS): protects neuron and controlsblood brain barrier and blood flow 2) microglia (CNS): mediate immune responses in the CNS by acting as macrophages 3) ependymal cells (CNS): forms a barrier between cerebral spinal fluid and neural tissue. cilia helps move fluid. 4) oligodendrocytes (CNS): forms myelin sheath 5) satellite cells (PNS): acts similarly to astrocytes 6) schwann cells (PNS): (similar in function to oligodendrocytes) but forms internode region and can repair nerve fibers

In order for you to realize there has been a sensory change, it has to reach the __________ level of processing. circuit receptor abstraction perceptual

?

Ruffini endings

Bulbous corpuscle

Which of the receptor types pictured functions exclusively as a proprioceptor?

B. Proprioceptors are sensitive to stimuli associated with body movements. The muscle spindle shown in B is responsive to muscle stretch.

True

Drooping of the upper eyelid, and double vision are potential symptoms of damage to the oculomotor nerve.

A dermatome represents the motor innervation of muscles in that area. True False

False Dermatomes represent sensory innervations of the underlying skin.

Describe cranial nerve III: The Oculomotor Nerves

Fibers extend from the ventral midbrain through the superior orbital fissures to the extrinsic eye muscles Functions in raising the eyelid, directing the eyeball, constricting the iris (parasympathetic), and controlling lens shape

Describe cranial nerve IV: The Trochlear Nerves

Fibers from the dorsal midbrain enter the orbits via the superior orbital fissures to innervate the superior oblique muscle Primarily a motor nerve that directs the eyeball

Describe cranial nerve VI: The Abducens Nerves

Fibers from the inferior pons enter the orbits via the superior orbital fissures Primarily a motor, innervating the lateral rectus muscle

Describe cranial nerve VII: The Facial Nerves

Fibers from the pons travel through the internal acoustic meatuses, and emerge through the stylomastoid foramina to the lateral aspect of the face Chief MOTOR nerves of the face with 5 major branches MOTOR functions include facial expression, parasympathetic impulses to lacrimal and salivary glands (tears & salivation) SENSORY function (taste) from the anterior two-thirds of the tongue

Describe the rami of spinal nerves

Each spinal nerve branches into mixed rami: - Dorsal ramus - Larger ventral ramus - Meningeal branch - Rami communicantes (autonomic pathways) join to the ventral rami in the thoracic region

Describe Roots of Spinal Nerves

Each spinal nerve connects to the spinal cord via two roots 1. Ventral roots - Contain motor (efferent) fibers from the ventral horn motor neurons - Fibers innervate skeletal muscles 2. Dorsal roots - Contain sensory (afferent) fibers from sensory neurons in the dorsal root ganglia - Conduct impulses from peripheral receptors

What is the structural and functional relationship between spinal nerves, skeletal muscles and dermatomes?

Each spinal nerve provides the sensory and motor supply of an adjacent muscle mass (destined to become skeletal muscles) and the cutaneous supply of a dermatome (skin segment).

Bill is a mechanic that works with vibrating tools. He also exerts force on his wrists when twisting wrenches and screws. Bill has a tingling sensation in the lateral portion of his hand. The doctor suspects carpal tunnel syndrome. Which of the following test might the doctor try on her patient?

Have bill grip an object with his thumb and index finger and try to pull the object away.

Crossed-extensor

Consists of an ipsilateral withdrawal reflex and a contralateral extensor reflex; important in maintaining balance.

Describe Ganglia

Contain neuron cell bodies associated with nerves Dorsal root ganglia (sensory, somatic) (Chapter 12) Autonomic ganglia (motor, visceral) (Chapter 14)

Describe the structure of a nerve

Cordlike organ of the PNS Bundle of myelinated and unmyelinated peripheral axons enclosed by connective tissue

How are most nerves classified?

Most nerves are mixtures of afferent and efferent fibers and somatic and autonomic (visceral) fibers Pure sensory (afferent) or motor (efferent) nerves are rare Peripheral nerves classified as cranial or spinal nerves

Neurons found in ventral horn.

Motor

B) involuntary, yet may be modified by learned behavior

Inborn or intrinsic reflexes are ________. A) rapid, predictable, and can be learned responses B) involuntary, yet may be modified by learned behavior C) autonomic only D) always mediated by the brain

Projection level

Includes cortical and brain stem motor areas.

Differentiate between ipsilateral and contralateral reflexes.

Ipsilateral reflexes involve a reflex initiated on and affecting the same side of the body; contralateral reflexes involve a reflex that is initiated on one side of the body and affects the other side.

The brain stem includes the ___.

Pons

In 1962 a boy playing in a train yard fell under a train. His right arm was cut off cleanly by the train wheel. Surgeons reattached the arm, sewing nerves and vessels back together. The boy was told he should eventually regain the use of his arm, but that it would never be strong enough to pitch a baseball. Explain why full recovery of strength was unlikely.

Precise realignment of cut, regenerated axons with their former effector targets is highly unlikely. Coordination between nerve and muscle will have to be relearned. Additionally, not all damaged fibers regenerate.

Stretch

Prevents muscle overstretching and maintains muscle tone.

Meissner corpuscle

Tactile corpuscle.

Precommand level

The cerebellum and basal nuclei.

If the ventral root of a spinal nerve were cut, what would be the result in the tissue or region that nerve supplies?

a complete loss of voluntary movement

B) accessory

The cranial nerve with a cervical origin (spinal cord) is the ________. A) hypoglossal B) accessory C) vagus D) glossopharyngeal

Mr. Frank, a former stroke victim who had made a remarkable recovery, suddenly began to have problems reading. He complained of seeing double and also had problems navigating steps. He was unable to move his left eye downward and laterally. What cranial nerve was the site of lesion? (Right or left?)

The left trochlear nerve (IV), which innervates the superior oblique muscle responsible for this action.

Segmental level

The neural machinery of the spinal cord, including spinal cord circuits.

Lumbar plexus

The obturator and femoral nerves branch from this plexus.

Describe cranial nerve X: The Vagus Nerves

The only cranial nerves that extend beyond the head and neck region Fibers from the medulla exit the skull via the jugular foramen Most MOTOR fibers are parasympathetic fibers that help regulate the activities of the heart, lungs, and abdominal viscera SENSORY fibers carry impulses from thoracic and abdominal viscera, baroreceptors, chemoreceptors, and taste buds of posterior tongue and pharynx

An emergency medical technician is examining a trauma victim by shining a pen light into her patient's eye. She records the reactivity of the patents pupils as they constrict when stimulated by the light. This test supports which of the following?

The patient has function of the oculomotor nerve (III).

Sacral plexus

The phrenic nerve branches from this plexus.

C) tibial

The posterior side of the thigh, leg, and foot is served by the ________ nerve. A) obturator B) common fibular C) tibial D) femoral

vagus nerve

The vagus nerve (cranial nerve X) is a component of the parasympathetic cranial outflow and controls normal function of organs of the thoracic and upper abdominal cavities.

Irritation of the phrenic nerve may cause diaphragm spasms called hiccups.

True

The craniosacral division is another name for the parasympathetic division.

True

The sensory division of the PNS is also known as the efferent division. True False

True All of the sensory information is considered afferent, or flowing toward the CNS. Efferent means "from the CNS."

Abducens

Turns the eyeball laterally.

Why might an individual experience the phenomenon known as "referred pain"?

Visceral pain afferents travel along the same pathways as somatic pain fibers.

A) median

Which nerve is compressed in carpal tunnel syndrome? A) median B) axillary C) radial D) ulnar

C) The stimulus energy must be converted into the energy of a graded potential called a transduction potential.

Which of the following is an incorrect statement regarding the occurrence of a sensation? A) The stimulus energy must match the specificity of the receptor. B) The stimulus energy must occur within the receptor's receptive field. C) The stimulus energy must be converted into the energy of a graded potential called a transduction potential. D) A generator potential in the associated sensory neuron must reach threshold.

A) Action potential frequency is increased as stimulus' strength increases.

Which of the following is the best explanation of how a stimulus' strength is transmitted to the central nervous system from sensory nerves? A) Action potential frequency is increased as stimulus' strength increases. B) An action potential will increase in strength as stimulus's strength increases. C) More than one type of receptor will respond to larger stimulus. D) Action potentials as well as graded potentials are sent to the central nervous system when stimulus strength increases.

B) phrenic

Which of the following nerves does not arise from the brachial plexus? A) median B) phrenic C) radial D) ulnar

C) nociceptors

Which receptors adapt most slowly? A) smell receptors B) pressure receptors C) nociceptors D) touch receptors

Bell's palsy is ________.

characterized by paralysis of facial muscles

Three main levels of neural integration operate in the somatosensory system. Which level involves the spinal cord?

circuit level Processing at the circuit level involves the transmission of action potentials along ascending pathways. These pathways deliver impulses to the appropriate regions of the cerebral cortex for localization and perception of the stimulus.

Three main levels of neural integration operate in the somatosensory system. Which level involves the spinal cord? perceptual level circuit level integrative level receptor level

circuit level Processing at the circuit level involves the transmission of action potentials along ascending pathways. These pathways deliver impulses to the appropriate regions of the cerebral cortex for localization and perception of the stimulus.

The sciatic nerve is a combination of which two nerves?

common fibular and tibial

Which reflex has a contralateral component? flexor tendon crossed-extensor stretch

crossed-extensor The crossed-extensor reflex activates opposing actions in the opposite limb.

Which reflex is triggered when a stranger suddenly grasps your arm?

crossed-extensor reflex The grasped arm is withdrawn (via the flexor reflex) as the opposite arm pushes you away from the attacker (via crossed-extensor reflex). As in this scenario, the crossed-extensor reflex often accompanies the flexor reflex.

In carpal tunnel syndrome, there may be tingling and numbness in the thumb due to compression of the ______.

cutaneous branches of the median nerve

In carpal tunnel syndrome, there may be tingling and numbness in the thumb due to compression of the ______.

cutaneous branches of the median nerve The median nerve cutaneous branch carries sensory input from the skin of the lateral 2/3rds of the hand, palm side and dorsum of fingers 2-3.

The phrenic nerve serves the __________.

diaphragm

Which of the following is NOT associated with the parasympathetic division of the autonomic nervous system (ANS)?

emergency action

Which connective tissue layer directly surrounds every axon in a nerve?

endoneurium Within a nerve, each axon is surrounded by endoneurium, a delicate layer of loose connective tissue that also encloses the fiber's associated Schwann cells.

Which connective tissue layer directly surrounds every axon in a nerve? endomysium perineurium endoneurium epineurium

endoneurium Within a nerve, each axon is surrounded by endoneurium, a delicate layer of loose connective tissue that also encloses the fiber's associated Schwann cells.

In a crossed-extensor reflex, if the right arm was grabbed it would flex and the left arm would ________.

extend

A patient who received a blow to the side of the skull exhibits the following signs and symptoms on that side of the face: he is unable to close his eye, and the corner of his mouth droops. Which cranial nerve has been damaged?

facial

A major nerve of the lumbar plexus is the

femoral

A major nerve of the lumbar plexus is the ________.

femoral

Damage to the ulnar nerve could result in the inability to ______.

flex the wrist

Damage to the ulnar nerve could result in the inability to ______.

flex the wrist One action of the flexor carpi ulnaris muscle is wrist flexion. This muscle is innervated by the ulnar nerve.

Which of the receptor types might function as a nociceptor? free nerve endings of sensory neurons

free nerve endings of sensory neurons Nociceptors respond to potentially damaging stimuli like noxious chemicals and extremes of temperature or pH. Nociceptors are generally associated with free nerve endings of specialized neurons.

turns the eyeball laterally

abducens

Which of the following cranial nerves carries only motor information? abducens olfactory optic trigeminal

abducens The abducens carries efferent (motor) signals to the extrinsic eye muscle that abducts the eye (turns it laterally).

Which of the following cranial nerves carries only motor information?

abducens nerve The abducens carries efferent (motor) signals to the extrinsic eye muscle that abducts the eye (turns it laterally).

formed by the union of a cranial and a spinal root

accessory

Which target organ is NOT affected by the parasympathetic division of the autonomic nervous system (ANS)?

adrenal medulla

Nerves that carry impulses toward the CNS only are ________.

afferent nerves

site of efferent soma

anterior horn

What type of nerve fibers are found in the ventral ramus of a spinal nerve?

both sensory and motor

striking the funny bone may cause injury to a nerve of this plexus

brachial

trauma to a nerve of this plexus may cause wrist drop

brachial

The majority of the cranial nerves attach to the __________.

brain stem

Which of the following is NOT required for a reflex arc?

interneurons

Which of the following is NOT required for a reflex arc? motor neurons sensory neurons interneurons receptors

interneurons In many cases, the interneurons are the integration point for reflexes, but they are not necessary.

Inborn or intrinsic reflexes are ________.

involuntary, yet may be modified by learned behavior

Which type of sensory receptor allows us to feel an insect landing on our skin?

mechanoreceptor Mechanoreceptors respond to mechanical force such as pressure, vibration, stretch, and touch, allowing us to feel an insect landing on our skin.

Tactile sensation is a combination of touch, pressure, stretch and vibration. Which of the following is most likely the receptor type that senses tactile stimulation?

mechanoreceptors

Which nerve is compressed in carpal tunnel syndrome?

median

In carpal tunnel syndrome, the __________ is compressed.

median nerve

In carpal tunnel syndrome, the __________ is compressed. radial nerve musculocutaneous nerve axillary nerve median nerve

median nerve

Hiccups could occur if there was irritation or damage to the ______.

motor branches of ventral rami associated with the C3-C5 region of the spinal cord

Hiccups could occur if there was irritation or damage to the ______.

motor branches of ventral rami associated with the C3-C5 region of the spinal cord The ventral rami of spinal nerves contain motor fibers that originate from neuron soma found in the spinal cord. These efferent pathways activate the contraction of skeletal muscle. The phrenic nerve branches from this region and innervates the skeletal muscle of the diaphragm.

Spinal nerves are all classified as __________. sensory nerves afferent nerves mixed nerves motor nerves

mixed nerves

most postganglionic sympathetic

norepinphrine

Which of the following cranial nerves carries only sensory information?

olfactory

receptors located in epithelium of the nasal cavity

olfactory

Segmental refers to which level of motor control?

spinal cord

The brachial plexus can be palpated at the lower lateral border of the sternocleidomastoid muscle. Injury to the brachial plexus could cause weakness or paralysis to all of the following EXCEPT the ______.

sternocleidomastoid muscle The sternocleidomastoid muscle is innervated by the accessory cranial nerve and branches of cervical spinal nerves C2 and C3.

The knee-jerk reflex is an example of a _________

stretch reflex

Which reflex requires gamma motor neurons to set the length of the muscle?

stretch reflex

auditory area

temporal

All processing at the circuit level going up to the perceptual level must synapse in the ________.

thalamus

a major relay station for sensory information ascending to primary sensory areas of the cerebral cortex

thalamus

gateway to the cerebrum

thalamus

In order for you to realize there has been a sensory change, it has to reach the __________ level of processing.

perceptual

Which of the following does NOT occur as people age?

peripheral nerves die off

Which of the following nerves does not arise from the brachial plexus?

phrenic

site of sensory soma

posterior horn

the cerebellum and basal nuclei

pre command

the brain area associates experiences necessary for the production of abstract ideas, judgment, and conscience

prefrontal

the axons from the area form the major pyramidal tracts

primary motor cortex

We can touch our finger to our nose while our eyes are closed in part because we can sense the position and movement of our joints as well as the length of stretch in our muscles. These sensations create awareness of our body's positioning. The following receptors are most likely responsible for this ability.

proprioceptors

site of axons and afferent neurons

ganglion

Use as refrence

http://www.easynotecards.com/print_list/63770

Which of the following is the correct simple spinal reflex arc?

receptor, afferent neuron, integration center, efferent neuron, effector

The letter E would represent which of the following muscles during the knee-jerk reflex?

rectus femoris The rectus femoris is one of the quadriceps. Stimulation of muscle spindles by tapping of the patellar ligament results in the reflex contraction of these muscles and the extension of the knee.

PNS Nerve Fibers

regenerate because of the actions of Schwann cells.

Which of these activities is most likely driven by parasympathetic innervation?

resting and digesting

Which of the following carries no sensory information?

hypoglossal nerve

Which of the following carries no sensory information? trigeminal nerve vestibulocochlear nerve hypoglossal nerve optic nerve

hypoglossal nerve The hypoglossal nerve carries motor commands to the tongue.

Emotions influence autonomic reactions primarily through integration in the ________.

hypothalamus

this area is the main visceral control center of the body

hypothalamus

visercal command center

hypothalamus

Starting at the spinal cord, the subdivisions of the brachial plexus are (in order):

roots, trunks, divisions, and cords

Gluteal innervation comes from which plexus?

sacral

a fall or improper administration of an injection to the buttocks may injure a nerve of this plexus

sacral

Spinal nerves exiting the cord from the level of L4 to S4 form the

sacral plexus

Spinal nerves exiting the cord from the level of L4 to S4 form the ________.

sacral plexus

The thickest and longest nerve in the body is the __________.

sciatic nerve

A fall or an improperly delivered gluteal injection could result in ________.

sciatica

central pattern generators

segmental

the neural machinery of the spinal cord, including spinal cord circuits

segmental

Which of the following lists the hierarchy of motor control from lowest to highest level of control?

segmental level, projection level, precommand level The hierarchy of motor control from lowest to highest level of control is segmental level, projection level, and precommand level.

Which plexus does NOT receive innervation from the vagus nerve?

inferior hypogastric plexus

As a cook chops red onions he begins to tear up due to activation of the lacrimal gland. Which of the following nerves provided the stimulus?

the facial nerve (VII)

Injury to cervical vertebra C3-C4 is particularly problematic because ________.

the phrenic nerve that serves the diaphragm receives its fibers from here

A patient has lost vision on the left side of both eyes. The patient has likely suffered damage to ________.

the right optic tract

A patient has an injury of the spine and is now suffering from a loss of motor function in his right arm. However, he still has normal sensory function in the arm. Based on this information it is likely that the patient has nervous tissue damage located at ________.

the ventral root located at one or more of the cervical vertebra

Dermatome maps are useful to clinicians because ________.

they can help pinpoint the location of spinal injury

Outflow of the sympathetic division occurs from which regions of the CNS?

thoracic and lumbar

The posterior side of the thigh, leg, and foot is served by the ________ nerve.

tibial

involved in movement of the digestive track

vagus

longest cranial nerve

vagus

all but T2-T12 branch and form nerve plexuses

ventral ramus

supplies all of the body except the posterior

ventral ramus

supplies each muscle with fibers from more than one nerve

ventral ramus

Problems in balance may follow trauma to which nerve?

vestibulocochlear

Which cranial nerve transmits information about our sense of equilibrium?

vestibulocochlear

damage to this nerve would cause dizziness, nausea, and loss of balance

vestibulocochlear

serves the senses of hearing and equilibrium

vestibulocochlear

horn containing autonomic neurons

lateral horn

Segmental level

Central pattern generators.

CNS nerve fibers regenerate because of the actions of Schwann cells.

FALSE

A reflex that causes muscle relaxation and lengthening in response to muscle tension is called a ________.

Golgi tendon reflex

Vagus

Helps to regulate blood pressure and digestion.

The Vagus Nerve (X) ____.

Innervates respiratory, cardiovascular and digestive muscles.

Describe sensory integration

Input comes from exteroceptors, proprioceptors, and interoceptors Input is relayed toward the head, but is processed along the way

Projection level

Intermediate relay for incoming and outgoing neurons.

Pacinian corpuscle

Lamellar corpuscle

What regions of our brain controls our emotions?

Limbic System

The following characteristics of neurons are true EXCEPT _____.

Low metabolic rate

Which type of sensory receptor allows us to feel an insect landing on our skin?

Mechanoreceptor (pressure, vibration, stretch, and touch)

What are meninges? List and describe the three meninges in the Central Nervous System.

Meninges are protective layers to the brain and spinal cord 1) Dura mater: outermost layer that is attached to periosteum 2) Pia mater: is the innermost layer which is directly attached to the brain 3) Arachnoid mater: middle later which is loosely bound to the brain and separated from the pia mater by the subarachnoid space which holds CSF

The Red Nucleus is located in the ____.

Midbrain

The resting potential of a cell is when the inside is _____ compared to the outside of the cell.

Negative

Which of the following is NOT correct concerning nerves?

Nerves are collection of axons of either sensory or motor neurons but not both

Which of the following is NOT correct concerning nerves?

Nerves are collection of axons of either sensory or motor neurons but not both.

A) afferent nerves

Nerves that carry impulses toward the CNS only are ________. A) afferent nerves B) efferent nerves C) motor nerves D) mixed nerves

__________ are receptors that can respond to painful stimuli.

Nociceptors

__________ are receptors that can respond to painful stimuli. Mechanoreceptors Chemoreceptors Nociceptors Photoreceptors

Nociceptors

__________ are receptors that can respond to painful stimuli.

Nociceptors Respond to potentially damaging stimuli that result in pain.

Gaps in the myelin sheath of PNS nerves are called ____.

Nodes of Ranvier

Everyday sympathetic tone ____.

Partially constricts the body's blood vessels

What is the functional relationship of the peripheral nervous system to the central nervous system?

The PNS enables the CNS to receive information and carry out its decisions.

List the structural components of the peripheral nervous system and describe the function of each component.

The PNS includes all nervous tissue outside the CNS and consists of the sensory receptors that detect specific stimuli, the peripheral nerves (cranial or spinal) that conduct impulses to and from the CNS, the ganglia that contain synapses or cell bodies outside the CNS and motor nerve endings that innervate effector organs.

Define PNS and list major components

The Peripheral Nervous System (PNS) is composed of all neural structures outside the brain: - Sensory receptors - Peripheral nerves and associated ganglia - Motor endings

Define Dermatome

The area of skin innervated by the cutaneous branches of a single spinal nerve (most dermatomes overlap, so destruction of a single spinal nerve will not cause complete numbness)

Once a preganglionic axon reaches a trunk ganglion, one of three things can happen to the axon. Which of the following is NOT do what?

The axon can course back into the spinal cord to synapse with preganglionic neurons in a different spinal segment.

Choose the FALSE statement about nerves.

The majority of a nerve's bulk is due to axons

True

The meningeal branch of a spinal nerve actually reenters the vertebral canal to innervate the meninges and blood vessels.

True

The musculocutaneous nerve is a major nerve of the brachial plexus.

D) common fibular and tibial

The sciatic nerve is a combination of which two nerves? A) pudendal and posterior femoral cutaneous B) posterior femoral cutaneous and tibial C) pudendal and common fibular D) common fibular and tibial

Which of the following is an incorrect statement regarding the occurrence of a sensation?

The stimulus energy must be converted into the energy of a graded potential called a transduction potential.

white rami communicantes

The white rami communicantes connect preganglionic fibers from the spinal cord to the sympathetic trunk.

Describe Spinal Nerves

These are only somatic, and every one is mixed: both sensory and motor The 31 pairs are named according to their point of issue from the spinal cord 8 cervical (C1-C8) 12 thoracic (T1-T12) 5 Lumbar (L1-L5) 5 Sacral (S1-S5) 1 Coccygeal (C0)

__________ do NOT exhibit the property of adaptation. Phasic receptors Sensory receptors Tonic receptors Photoreceptors

Tonic receptors

_____________________ do NOT exhibit the property of adaptation

Tonic receptors

A dendrite conducts graded potentials _____ the cell body.

Toward

B) stimulus energy into energy of a graded potential

Transduction refers to conversion of ________. A) presynaptic nerve impulses to postsynaptic nerve impulses B) stimulus energy into energy of a graded potential C) receptor energy to stimulus energy D) afferent impulses to efferent impulses

Brachial plexus

Trauma to a nerve of this plexus may cause wrist drop.

A dermatome represents the motor innervation of muscles in that area.

True

supplies the posterior body trunk

dorsal ramus

The flexor muscles in the anterior arm (biceps brachii and brachialis) are innervated by what nerve?

musculocutaneous

If someone spills very hot coffee (200 0F) on their skin, they will likely perceive much pain. Which of the following receptor types is causing this sensation?

nociceptors

Which of the following is NOT used to classify receptors?

the number of dendritic endings present

Which of the following is NOT used to classify sensory receptors?

the number of dendritic endings present

damage to this nerve would keep the eye from rotating inferolaterally

trochlear

innervates the superior oblique muscle

trochlear

CNS nerve fibers lack the intrinsic capacity to regenerate, while PNS nerve fibers are able to regenerate.

true

Striking the "funny bone" is actually stimulation of (or injury to) the ________.

ulnar nerve

the phrenic nerve branches from this plexus

cervical

THe phrenic nerve is a branch from the

cervical plexus

The phrenic nerve is a branch from the __________.

cervical plexus

What reaction would occur if the muscle labeled E were suddenly pulled longer?

the contraction of muscle E A sudden stretch in a muscle leads to the contraction of the stretched muscle (E) and the relaxation of the antagonistic muscle (G).

Which of the following is NOT used to classify sensory receptors? structural complexity location the type of stimulus they detect the number of dendritic endings present

the number of dendritic endings present

False

Reciprocal inhibition means that while one sensory nerve is stimulated, another sensory neuron for synergistic muscles in the same area is inhibited and cannot respond.

Mr Jake was admitted to the hospital with excruciating pain in his left shoulder and arm. He was found to have suffered a heart attack. Explain the phenomenon of referred pain as exhibited by Mr Jake.

Referred pain occurs because visceral and somatic pain fibers travel along the same neural pathways. Mr. Jake felt pain in his left arm because the heart, located on the left side of the thoracic cavity, would share pathways with the left arm.

What clinical information can be gained by conducting somatic reflex tests?

Reflex tests assess the condition of the nervous system. Exaggerated, distorted, or absent reflexes indicate degeneration or pathology of specific regions of the nervous system often before other signs are apparent.

C) is prevented due to growth-inhibiting proteins of oligodendrocytes

Regeneration within the CNS ________. A) is more successful than with the PNS B) typically allows axonal sprouting of 20 mm C) is prevented due to growth-inhibiting proteins of oligodendrocytes D) is promoted by growth inhibitors and glial scars

Differentiate clearly between sensation and perception.

Sensation is simply the awareness of a stimulus, whereas perception also understands the meaning of the stimulus.

Define Sensation

Sensation: the awareness of changes in the internal and external environment

Which cells of the PNS trigger the feeling of a 'belly ache'?

Sensory neurons

Vestibulocochlear

Serves the senses of hearing and equilibrium.

Anterior arm muscles that flex the forearm. (A) brachial (B) cervical (C) lumbar (D) sacral (1) common fibular (2) femoral (3) median (4) musculocutaneous (5) obturator (6) phrenic (7) radial (8) tibial (9) ulnar

(A) brachial (4) musculocutaneous

Muscles that extend the wrist and digits. (A) brachial (B) cervical (C) lumbar (D) sacral (1) common fibular (2) femoral (3) median (4) musculocutaneous (5) obturator (6) phrenic (7) radial (8) tibial (9) ulnar

(A) brachial (7) radial

Skin and extensor muscles of the posterior arm. (A) brachial (B) cervical (C) lumbar (D) sacral (1) common fibular (2) femoral (3) median (4) musculocutaneous (5) obturator (6) phrenic (7) radial (8) tibial (9) ulnar

(A) brachial (7) radial

Slowly adapting deep pressure receptor. (A) bulbous corpuscles (B) tendon organ (C) muscle spindle (D) free nerve endings (E) lamellar corpuscle (F) tactile corpuscle

(A) bulbous corpuscle

You re enjoying an ice cream cone. (A) exteroceptor (B) interoceptor (C) proprioceptor (1) chemoreceptor (2) mechanoreceptor (3) nociceptor (4) photoreceptor (5) thermoreceptor

(A) exteroceptor (1) chemoreceptor (5) thermoreceptor

You have just scalded yourself with hot coffee. (A) exteroceptor (B) interoceptor (C) proprioceptor (1) chemoreceptor (2) mechanoreceptor (3) nociceptor (4) photoreceptor (5) thermoreceptor

(A) exteroceptor (3) nociceptor (5) thermoreceptor

You feel uncomfortable after a large meal. (A) exteroceptor (B) interoceptor (C) proprioceptor (1) chemoreceptor (2) mechanoreceptor (3) nociceptor (4) photoreceptor (5) thermoreceptor

(B) interoceptor (2) mechanoreceptor

The large onion shaped receptors that are found deep in the dermis and in subcutaneous tissue and that respond to deep pressure are... (A) tactile discs (B) lamellar corpuscles (C) free nerve endings (D) muscle spindles

(B) lamellar corpuscles

Contains receptor endings wrapped around thick collagen bundles. (A) bulbous corpuscles (B) tendon organ (C) muscle spindle (D) free nerve endings (E) lamellar corpuscle (F) tactile corpuscle

(B) tendon organ

Impaired in Belle's palsy. (A) abducens (B) accessory (C) facial (D) glossopharyngeal (E) hypoglossal (F) oculomotor (G) olfactory (H) optic (I) trigeminal (J) trochlear (K) vagus (L) vestibulocochlear

(C) facial

Contain parasympathetic motor fibers (4 nerves). (A) abducens (B) accessory (C) facial (D) glossopharyngeal (E) hypoglossal (F) oculomotor (G) olfactory (H) optic (I) trigeminal (J) trochlear (K) vagus (L) vestibulocochlear

(C) facial (D) glossopharyngeal (F) oculomotor (K) vagus

Anterior thigh muscles. (A) brachial (B) cervical (C) lumbar (D) sacral (1) common fibular (2) femoral (3) median (4) musculocutaneous (5) obturator (6) phrenic (7) radial (8) tibial (9) ulnar

(C) lumbar (2) femoral

Medial thigh muscles. (A) brachial (B) cervical (C) lumbar (D) sacral (1) common fibular (2) femoral (3) median (4) musculocutaneous (5) obturator (6) phrenic (7) radial (8) tibial (9) ulnar

(C) lumbar (5) obturator

Contains intrafusal fibers and anulospiral and flower spray endings. (A) bulbous corpuscles (B) tendon organ (C) muscle spindle (D) free nerve endings (E) lamellar corpuscle (F) tactile corpuscle

(C) muscle spindle

The connective tissue sheath that surrounds a fascicle of nerve fibers is the... (A) epineurium (B) endoneurium (C) perineurium (D) epimysium

(C) perineurium

You are in a complete dark room and reaching toward the light switch. (A) exteroceptor (B) interoceptor (C) proprioceptor (1) chemoreceptor (2) mechanoreceptor (3) nociceptor (4) photoreceptor (5) thermoreceptor

(C) proprioceptor (2) mechanoreceptor

Pain, itch, and temperature receptors. (A) bulbous corpuscles (B) tendon organ (C) muscle spindle (D) free nerve endings (E) lamellar corpuscle (F) tactile corpuscle

(D) free nerve endings

Posterior arm fibularis muscles, tibialis anterior and toe extensors. (A) brachial (B) cervical (C) lumbar (D) sacral (1) common fibular (2) femoral (3) median (4) musculocutaneous (5) obturator (6) phrenic (7) radial (8) tibial (9) ulnar

(D) sacral (1) common fibular

Muscles for the posterior leg. (A) brachial (B) cervical (C) lumbar (D) sacral (1) common fibular (2) femoral (3) median (4) musculocutaneous (5) obturator (6) phrenic (7) radial (8) tibial (9) ulnar

(D) sacral (8) tibial

The aspect of sensory perception by which the cerebral cortex identifies the site or pattern of stimulation is...... (A) perceptual detection (B) feature abstraction (C) pattern recognition (D) spatial discrimination

(D) spatial discrimination

Dorsal root ganglia contain..... (A) cell bodies of somatic motor neurons (B) axon terminals of somatic motor neurons (C) cell bodies of automatic motor neurons (D) axon terminals of sensory neurons (E) cell bodies of sensory neurons

(E) cell bodies of sensory neurons

Serves the tongue muscles. (A) abducens (B) accessory (C) facial (D) glossopharyngeal (E) hypoglossal (F) oculomotor (G) olfactory (H) optic (I) trigeminal (J) trochlear (K) vagus (L) vestibulocochlear

(E) hypoglossal

Discriminative touch receptors in hairless skin (fingertips). (A) bulbous corpuscles (B) tendon organ (C) muscle spindle (D) free nerve endings (E) lamellar corpuscle (F) tactile corpuscle

(F) tactile corpuscle

Allows you to chew your food. (A) abducens (B) accessory (C) facial (D) glossopharyngeal (E) hypoglossal (F) oculomotor (G) olfactory (H) optic (I) trigeminal (J) trochlear (K) vagus (L) vestibulocochlear

(I) trigeminal

The major sensory nerve of the face. (A) abducens (B) accessory (C) facial (D) glossopharyngeal (E) hypoglossal (F) oculomotor (G) olfactory (H) optic (I) trigeminal (J) trochlear (K) vagus (L) vestibulocochlear

(I) trigeminal

Helps regulate heart activity. (A) abducens (B) accessory (C) facial (D) glossopharyngeal (E) hypoglossal (F) oculomotor (G) olfactory (H) optic (I) trigeminal (J) trochlear (K) vagus (L) vestibulocochlear

(K) vagus

Helps you hear and maintain your balance. (A) abducens (B) accessory (C) facial (D) glossopharyngeal (E) hypoglossal (F) oculomotor (G) olfactory (H) optic (I) trigeminal (J) trochlear (K) vagus (L) vestibulocochlear

(L) vestibulocochlear

Compare and contrast the somatic and autonomic nervous systems in terms or neurons (pre- and post-, and their structure), neurotransmitters released, effectors organs, and the overall effect of the system.

(Look @ Figure)

What is myelin? How does the myelination process differ in the CNS and PNS.

- Myelin is a white, fatty, insulating, material that makes up plasma membrane wrappings of schwann cells and oligondendrosites. --> In the CNS oligondendrosites have a plasma membrane wrapping that is a flappy extension which can cover various nerve fiber axons. The PNS has shwann cells where each wrapped shwann cell makes up internode region. It also can repair nerve fibers while CNS cannot.

How are the axons and dendrites alike? In what ways (structurally and functionally) do they differ?

--> Dendrites are short, while axons are longer. The dendrites take information from the pre-synaptic cell to the cell body. Axons take information away from the cell body and bring it towards the post-synaptic cell.

What specifically determines whether an EPSP or IPSP will be generated at the postsynaptic membrane?

--> Determination whether an EPSP or IPSP will be generated at the postsynaptic membrane depends on where the NT binds.

Homework Questions:

---------------

A brain surgeon is about to make an incision. Name all the tissue layers that she cuts through from the skin to the brain.

---> For a surgeon to make an incision, she must cut through 6 layers of to reach the brain and sequentially follows this order: skin, skull, dura matter, arachnoid membrane, pia matter, and the brain.

Explain how a crossed-extensor reflex exemplifies both serial and parallel processing.

--> A crossed-extensor reflex exemplifies both serial and parallel professing in that the simultaneous signals are being sent to both of the muscle groups. Both of these muscles groups being the flexors and the extensors contracting and relaxing. The serial processing in the activation of the interneurons turns the inhibition of the motor neurons.

Describe the events that must occur to generate an AP. Relate the sequence of changes in permeability to changes in the ion channels, and explain why the AP in an all-or-none phenomenon.

--> APs are electrical impulses that are carried along an axon, are generated in the axon hillock and travel down the axon. This then helps them reach their destination. The special thing about APs are that they are generated only by muscle cells and neurons and they do not decrease over time when it comes to strength. There is a change in the membrane potential when a neuron stimulates. This can be changed in the stimulus is applied to the -55mv threshold. Depolarization is self-generating to create an AP and this is an "all-or-none" phenomenon, therefore it is not generated if it doesn't reach its threshold. The depolarization phase includes Na+ permeability to increase so the voltage-gated Na+ can open and then makes the membrane less negative. The K+ gates are still closed at this time. The membrane is depolarized to +35 and then inactivation gates close. Therefore, the membrane permeability to Na+ declines to rest and then K+ gates open. K+ starts to diffuse out of the cell and the membrane become more negative. As the gates remain open for the K+, the excess amount of K+ diffuses out of the cell which causes a more negative state (hyperpolarization). This time creates a harder generation in action potential because of the negative state. The regulated gates for Na/K are closed, therefore cleaning to the Na/K pump to restore its ionic balance.

Local anesthetics block voltage gated Na+ channels. General anesthetics are thought to activate chemically gated Cl- channels, thereby rendering the nervous system quiescent while surgery is performed. What specific process do anesthetics impair, and how does this interfere with nerve impulse transmission?

--> Anesthetics impair specific processes such as impulse propagation. This interferes with nerve impulse transmission because there isn't enough Na+ entering the cell which means there is no minimum threshold causing no AP. Therefore, Cl- enters the cell, causes hyperpolarization and no AP.

Explain why damage to peripheral nerve fibers is often reversible whereas damage to CNS fibers rarely is.

--> Damage to peripheral nerve fibers is often reversible, whereas damage to CNS fibers rarely is because the CNS lacks a neurilemmal. The importance in the neurilemmal is that it plays a large role in fiber regeneration.

Five-year-old Amy wakes her parents up at 3am crying and complaining of a sore neck, severe headache, and feeling sick to her stomach. She has a temperature of 40°C and hides her eyes, saying that the lights are too bright. The emergency physician suspect meningitis and performs a lumbar tap. Using your knowledge of neuroanatomy, explain into which space and at what level of the vertebral column the needle will be inserted to perform this test. Which fluid is being obtained and why?

--> Due to Amy's symptoms and diagnosis of the homeostatic imbalance that is occurring within her body, she needs to endure a lumbar tap. Due to this information, it this procedure needs to be completed to diagnose her with meningitis specifically. A sample of cerebrospinal fluid needs to be obtained for this specific procedure. The needle must be inserted correctly, meaning inserted in the space of the subarachnoid space within the meningeal sac inferior to that point to ideally withdraw the fluid for testing. This is typically between the L1 and L2 regions of the spinal column or located around midrange of the L5 vertebral column region. The cerebrospinal fluid is obtained specifically for testing to determine if she does have the meningitis as it will enable the examining for microbe.

What is the homeostatic value of flexor reflexes?

--> Flexor reflexes help maintain homeostasis by permitting the body to make adjustments when need be.

As harry fell off a ladder, he grabbed a tree branch with his right hand, but unfortunately lost his grip and fell heavily to the ground. Days later, Harry complained that his upper limb was numb. What was damaged in his fall?

--> His brachial plexus was damaged in his fall. This occurred as he stopped his fall by grabbing the branch to help, but in the end caused this damage.

What clinical information can be gained by conducting somatic reflex tests.

--> Information such that somatic reflexes that are involved in various skeletal muscles of mammal body can be gained. These tests can tell a physician if the muscles are holding the skeleton together and if they are functioning properly or not.

Differentiate between ipsilateral and contralateral reflexes.

--> Ipsilateral reflexes are the sensory receptors and effectors that are associated on the same side of the body. Contralateral reflexes are the sensory receptors and effectors are on opposite sides of the body.

Robert, a brilliant computer analyst, suffered a blow to his anterior skill from a falling rock while mountain climbing. Shortly thereafter, it was obvious to his coworkers that his behavior had undergone a dramatic change. Although previously a smart dresser, he was not unkempt. One morning, he was observed defecting into the wastebasket. His supervisor ordered Robert to report to the company's doctor immediately. Which region of Robert's brain was affected by the cranial blow?

--> Most likely, Robert's frontal lobe of his brain has been affected by the cranial blow caused by a falling rock while mountain climbing. This can be known because the frontal lobe is extremely important in a human's cognitive skills. This includes emotional expression, problem solving, memory, language, etc. It's supposed to be the control panel of our personalities and the ability to act appropriately and communicate properly. Due to Robert's actions, it seems as if this would be the most reasonable region to be affected.

What is a reflex? List the 5 steps in the reflex pathway. Name the 2 types of reflexes, given an example of each.

--> Reflex: A rapid, predictable, involuntary motor response to a stimulus. --> Pathway: (1) receptor, (2) sensory neuron, (3) integration center, (4) motor neuron, (5) effector --> (1) Inborn (intrinsic) reflex: pulling hand away from stove --> (2) Learned (acquired) reflex: driving a car

how does the myelination process differ in the CNS and PNS?

--> Schwann cells are in the process of myelination in the PNS while oligodendrocytes myelinate in the CNS.

Differentiate clearly between sensation and perception.

--> Sensation and perception are completely different elements of how we process information. Sensation is just something that happens to you and how you feel it whereas perception is how your brain makes sense of the thing that happens to you. Sensation is the result of the body's senses such as it sensing something hot, cold, painful, etc. Our brains then convert many sensations into feelings and then this can express to others the feeling of emotion. Perception is how we see the world. This includes what can be seen, what isn't seen, and what you can see that's not there. This information is then processes and translated and interpreted.

What is the structural and functional relationship between spinal nerves, skeletal muscles, and dermatomes?

--> Spinal nerves have dermatomal distributions. Spinal nerves derive peripheral nerves that constrain fibers and the spinal nerves that enter a plexus contributes to the peripheral nerves. They provide sensory and motor supply to an adjacent muscle mass which are going to be skeletal muscles and the dermatomes. Dermatomes are the area of skin supplied by the sensory fibers of a single dorsal root. This is through the dorsal and ventral rami of the spinal cord. Dermatomes have little specific correspondence with the underlying muscles.

Which ANS fibers release acetylcholine? Which release norepinephrine?

--> The ANS fibers such as preganglionic fibers and postganglionic fibers of the parasympathetic division secrete acetylcholine. Some postganglionic sympathetic fibers secrete the acetylcholine too but only postganglionic fibers of sympathetic release norepinephrine.

Which area of the brain is most directly involved in mediating autonomic reflexes?

--> The area of the brain that is most directly involved in mediating autonomic reflexes is the reticular formation nuclei in the brain stem. These are particularly those in the medulla.

What constitutes the blood brain barrier?

--> The blood brain barrier is a protective mechanism of the brain. It's important for many reasons in protection of just the brain, no other body tissues. Importantly, there are seven key components that constitute the blood brain barrier: specialized basal membrane, enzymatic blood brain barrier controlling entry of neurotransmitters, pericytes, astrocytic end feet, Na transporters, Na-K ATPase, transport carriers for molecules such as glucose and amino acids, and the secondary transport for efflux from brain to blood. This important barrier reduces any exposure to harmful chemicals or agents that may cause any harm.

Describe the role of the cerebellum in maintain smooth, coordinated skeletal muscle activity.

--> The cerebellum is important because it provides smoot, coordinated skeletal muscle activity. In other words, this means that it provides the specific timing for certain movements and agility. It also regulates important patterns that need to be followed for our daily functions. Without cerebellum, we may be stumbling and falling all over the place as disruptions of this area can cause one to do so. It also plays an important role in controlling the rapid movements necessary for speech.

Mr. Miller is hospitalized for cardiac problems. Somehow, medical orders are mixed up and Mr. Miller is infused with a K+ enhanced intravenous solution meant for another patient who is taking potassium-wasting diuretics (i.e. drugs that cause excessive loss of potassium from the body in urine). Mr. Miller's potassium levels are normal before the IV is administered. What do you think will happen to Mr. Miller's resting membrane potentials? To his neuron's ability to generate APs?

--> The potassium enhanced solution would increase the concentration of the brain potassium ions. The positive charged potassium ions depolarize the resting membrane potential and they would want to flow to the brain. This would be due to the concentration gradient. The generation of APs would be increase due to the depolarization.

As the aroma of freshly brewed coffee drifted by dozing Henry's nose, his mouth started to water and his stomached begin to rumble. Explain his reactions in terms of ANS activity.

--> The smell is being used by the olfactory nerves. This is carried to the CNS as important information that needs to be responded to. This is known as an example of parasympathetic activation because it stimulates an increase in salivary gland secretion, mouth-watering. It also increases the secretory activity and the sound of the stomach rumbling.

When admitted to the emergency room, John was holding his right hand, which had a deep puncture hole in its palm. He explained that he had fallen on a nail while exploring a barn. John was given an antitetanus shot to prevent neural complications. Tetanus bacteria fester in deep, dark wounds, but how do their toxins travel in neural tissue?

--> Their toxins travel into the neural tissue because certain viruses and bacterial toxins that damage neural tissues are used to retrograde axonal transport between the cell bodies to move towards them.

Since all APs generated by a given nerve fiber have the same magnitude, how does the CNS "know" whether a stimulus is strong or weak?

--> There is a relationship between the frequency of AP and the strength of the stimulus; the higher the strength of stimulus, the higher the frequency.

Since at any moment a neuron is likely to have thousands of neurons releasing transmitters at its surface, how is neuronal activity (to fire or not to fire) determined?

--> This is based on an "all-or-nothing" basis. Once there is a stimulus reaches the threshold, the neuron will fire no matter what. If the input it receives is too weak to the threshold, the firing will not occur because it lacks those located area stimulations. If it receives opposite inputs, the same principle applies and the strongest stimulus will make it to the threshold to win.

In 1962 a boy playing in a train yard fell under a train. The train wheel cleanly cut off his right arm. Surgeons reattached the arm, sewing nerves and vessels back together. The boy was told he should eventually regain the use of his arm but that it would never be strong enough to pitch a baseball. Explain why full recovery of strength was unlikely.

--> This is because the precise realignment of cut, the regeneration of the specific axons is unlikely as their former effector targets are effected. This coordination would have to be relearned between nerve and muscles. Unfortunately, all of the damaged fibers will not all regenerate either.

Rochelle developed multiple sclerosis when she was 27. After eight years she had lost a good portion of her ability to control her skeletal muscles. How did this happen?

--> This is caused by the destroying of myelin sheaths in the CNS by the immune system. It is attacking the myelin proteins gradually which causes them to reduce to nonfunctional hardened lesions. This are known as scleroses. The successive gaps are excited more and more slowly which eventually causes impulse conductions to cease. Without the consistent innervation, a cause of weakness, clumsiness, and paralyzing symptoms can occur.

Mr. Frank, a former stroke victim who had made a remarkable recovery, suddenly began to have problems reading. He complained of seeing double and also had problems navigating steps. He was unable to move his left eye downward and laterally. What cranial nerve was the site of lesion? (Right or left?)

--> This would be his left trochlear nerve being the IV nerve. This is in which innervates the superior oblique muscle. This is responsible for this action causing his specific symptoms.

Tiffany, a 21-year-old college student, is having trouble sleeping, crying frequently, and having recurrent thoughts of suicide. An antidepressant is prescribed. Like many drugs, this antidepressant has anticholinergic side effects. What side effects might Tiffany experience in the first week of treatment?

--> Unfortunately, Tiffany may experience some light-headedness, blurred vision, maybe a dry mouth, constipation, and a difficulty in urinating or incontinence.

One of a group of rabbit hungers was accidentally sprayed with buckshot in both of his gluteal prominences. When his companions saw that he would survive, they laughed and joked about where he had been shot. They were horrified and ashamed a week later when it was announced that their friend would be permanently paralyzed and without sensation in both legs from the knee down, as well as on the back of his thighs. What had happened?

--> What had happened is that there must have been a severe damage by the shooting accident of the motor and sensory regions. This is because the loss follows the course of the sciatic nerves and their divisions would cause the issue.

[a] How is cerebrospinal fluid formed and drained? Describe its pathway wining and around the brain. [b] What happens if CSF does not drain properly? Why is this consequence more harmful in adults?

--> [a] Cerebrospinal fluid is formed and drained at a constant rate usually. If there is something that gets in the way of this occurring, such as a tumor or something, CSF accumulates and exerts a pressure on a brain which is not ideal. This condition is usually referred to as hydrocephalus. The pathway of the CSF can be described by the following 4 steps: (1) The choroid plexus of each ventricle produces CSF. (2) CSF flows through the ventricles and into the subarachnoid space via the median and lateral apertures. (3) CSF flows through the subarachnoid space. (4) CSF is absorbed into the dural venous sinuses via the arachnoid villi. --> [b] If this pathway is prohibited, the condition hydrocephalus occurs and can enlarge heads such as newborn babies because the skill bones did not have the chance to fuse together yet. This is a more serious matter in adult head simply because adult heads have completely formed, and the hydrocephalus is likely to damage the brain because the fluid compresses blood vessels and crushes the soft nervous tissue because of the skill being rigid and hard at this point in maturation.

[a] Define concussion and contusion. [b] Why does severe brain stem injury result in unconsciousness?

--> [a] Concussion is defined as an injury due to a broader scale brain trauma. It can effect one or many regions of the brain and can also range in severity and symptoms. A Contusion is defined as a localized injury, comparable to bruises that we get if we hit some part of our body. Just like other bruises, contusions can range from severity such that they can be extremely minor or life-threatening. --> [b] Severe brain stem injuries can result in unconsciousness because the brainstem could become compromised. As the brainstem specifically controls wakefulness and consciousness, the interruption that occurs can cause the unconsciousness. Certain phenomena such as a stroke or bleeding in a certain area be an example of these severe brain stem injuries which, can result in a large amount of pressure on the brainstem and will likely cause unconsciousness.

Name 5 functions of the hypothalamus.

1) Controls the autonomic nervous system (BP rate, force of heartbeat, digestive tract mobility, and pupil size) 2) Physical responses to emotions (limbic system): scallop example --> perception of pleasure, fear, rage, and in biological rhythms and drives 3) Regulates body temperature (sweating/shivering) 4) Regulates hunger in response to nutrient blood levels or hormones 5) Regulates water balance and thirst

Name & describe the functions of the cerebral five lobes.

1) Frontal Lobe: controls higher cognitive functioning such as decision making and judgment. 2) Parietal Lobe: controls sensations along with reading and writing 3) Occipital Lobe: vision 4) Temporal Lobe: controls auditory senses, word meaning, etc. 5) Insula: relates temperature, pain, and taste

List four sensory receptors and briefly describe their functions.

1) Mechanoreceptors: responds to touch, pressure, vibration, and stretch 2) Thermoreceptors: sensitive/responds to changes in temperature 3) Photoreceptors: responds to light energy 4) Chemoreceptors: respond to chemicals 5) Nociceptors: sensitive to pain-causing stimuli

The Diencephalon has three divisions, name the divisions and briefly their functions.

1) epithalamus: Has the pineal gland present which: secretes hormone melatonin (helps regulate with our sleep cycle & other) 2) thalamus: Relays and processes sensory information --> gateway to cerebral cortex 3) hypothalamus: Master gland of body: control center --> regulates pituitary gland

(1.) Define plexus. (2.) Indicate the spinal roots of origin of the four major nerve plexuses and name the general body regions served by each.

1. A plexus is a branching nerve network formed by roots from several spinal nerves that ensures that any damage to one nerve root will not result in total loss of innervation to that part of the body. 2. See picture for a sketch of where the four major nerve plexuses are located.

List and describe the five nerves of the brachial plexus (including the areas they innervate)

1. Axillary—innervates the deltoid, teres minor, and skin and joint capsule of the shoulder 2. Musculocutaneous—innervates the biceps brachii and brachialis and skin of lateral forearm 3. Median—innervates the skin, most flexors and pronators in the forearm (anterior side), and some intrinsic muscles of the hand such as skin of palm and first three fingers (this is the nerve involved in carpal tunnel syndrome) 4. Ulnar—supplies the flexor carpi ulnaris, part of the flexor digitorum profundus, most intrinsic muscles of the hand, and skin of medial aspect of hand (fingers 4 & 5) 5. Radial—innervates essentially all extensor muscles (posterior side), supinators, and posterior skin of limb

Central pattern generators (CPGs) are found at the segmental level of motor control. (1.) What is the job of the CPGs? (2.) What controls them and where is this control localized?

1. Central pattern generators (CPGs) control often repeated locomotion and motor activities. 2. The precommand center, the cerebellum and basal nuclei, modify and control the activity of the CPG circuits.

List and describe classifications of sensory receptors based on structure

1. Complex receptors (special sense organs):sense vision, hearing, equilibrium, smell, and taste (Chapter 15) 2. Simple receptors for general senses (dendritic nerve endings): for tactile sensations (touch, pressure, stretch, vibration), temperature, pain, and muscle sense. They can be unencapsulated (free) or encapsulated dendritic endings

List and describe types of sensory receptors with encapsulated dendritic endings

1. Meissner's (tactile) corpuscles—discriminative touch (found in hairless skin: nipples, fingertips, soles of feet, eyelids, external genitalia ) 2. Pacinian (lamellated) corpuscles—deep pressure and vibration (found in dermis, hypodermis, tendons, joints) 3. Ruffini endings—deep continuous pressure (found deep in dermis, hypodermis, and joint capsules) 4. Muscle spindles—muscle stretch (found in skeletal muscles, especially those of extremities) 5. Golgi tendon organs—stretch in tendons (found wrapped around tendons) 6. Joint kinesthetic receptors—stretch in articular capsules (found in joint capsules of synovial joints) ALL are mechanoreceptors

What are the two types of fibers in mixed nerves?

1. Somatic afferent and somatic efferent (skin, muscles, joints) 2. Visceral afferent and visceral efferent (internal organs)

B) the vestibulocochlear nerve (VIII)

A doctor asks her patient to follow the motion of her finger as she moves it up and down, left and right. Which of the following cranial nerves is not being tested? A) the abducens (VI) B) the vestibulocochlear nerve (VIII) C) the oculomotor nerve (III) D) the trochlear nerve (IV)

D) sciatica

A fall or an improperly delivered gluteal injection could result in ________. A) neurofibromatosis B) postpoliomyelitis muscular atrophy C) phantom limb pain D) sciatica

Sacral plexus

A fall or improper administration of an injection to the buttocks may injure a nerve of this plexus.

C) olfactory

A fracture of the ethmoid bone could result in damage to which cranial nerve? A) glossopharyngeal B) vagus C) olfactory D) accessory

A) femoral

A major nerve of the lumbar plexus is the ________. A) femoral B) iliohypogastric C) sciatic D) ilioinguinal

The synapse between which of the following two neurons is a part of a monosynaptic reflex arc?

A monosynaptic reflex arc includes a synapse between sensory and motor neurons without the involvement of an interneuron.

C) the ventral root located at one or more of the cervical vertebra

A patient has an injury of the spine and is now suffering from a loss of motor function in his right arm. However, he still has normal sensory function in the arm. Based on this information it is likely that the patient has nervous tissue damage located at ________. A) spinal nerves of the cervical vertebra B) the dorsal root located at one or more of the cervical vertebra C) the ventral root located at one or more of the cervical vertebra D) the dorsal rootlets located at one of the thoracic vertebra

B) the right optic tract

A patient has lost vision on the left side of both eyes. The patient has likely suffered damage to ________. A) the optic nerves B) the right optic tract C) the retinas of the eyes D) the optic chiasm

D) olfactory nerve (I)

A patient is suffering from the inability to distinguish various types of odors. This patient may have damage to which of the following? A) hypoglossal nerve (XII) B) vagus nerve (X) C) facial nerve (VIII) D) olfactory nerve (I)

Which of the receptor types might function as a nociceptor?

A) Nociceptors respond to potentially damaging stimuli like noxious chemicals and extremes of temperature or pH. Nociceptors are generally associated with free nerve endings of specialized neurons.

How quickly can a PNS axon regrow?

About 1.5 mm of growth per day

Cholinergic receptors bind _____.

Acetylcholine

Which of the following is the best explanation of how a stimulus' strength is transmitted to the central nervous system from sensory nerves?

Action potential frequency is increased as stimulus' strength increases.

Nerves that carry impulses toward the CNS only are

Afferent

Describe cranial nerve VIII: The Vestibulocochlear Nerves

Afferent fibers from the hearing receptors (cochlear division) and equilibrium receptors (vestibular division) pass from the inner ear through the internal acoustic meatuses, and enter the brain stem at the pons-medulla border MOSTLY sensory function; small motor component for adjustment of sensitivity of receptors

B) Schwann cells

After axonal injury, regeneration in peripheral nerves is guided by ________. A) Wallerian cells B) Schwann cells C) dendrites D) Golgi organs

Which of these receptor types functions as an exteroceptor? hair follicle receptors Lamellar (Pacinian) corpuscles Tactile (Merkel) discs All of the listed responses are correct

All of the listed responses are correct. All of the structures listed are sensitive to stimuli arising outside the body.

B) thalamus

All processing at the circuit level going up to the perceptual level must synapse in the ________. A) pons B) thalamus C) reticular formation D) medulla

Which is the only spinal nerve not participating in a dermatome?

All spinal nerves except C1 participate in dermatomes

Which ventral rami form nerve networks and what are they called?

All ventral rami except T2-T12 form interlacing nerve networks called plexuses (cervical, brachial, lumbar, and sacral)

B) The patient has function of the oculomotor nerve (III).

An emergency medical technician is examining a trauma victim by shining a pen light into her patient's eye. She records the reactivity of the patents pupils as they constrict when stimulated by the light. This test supports which of the following? A) The patient has suffered brain damage. B) The patient has function of the oculomotor nerve (III). C) The patient has lost function of the optic nerve (II). D) The patient has function of the trochlear nerve (IV).

The sensory division of the PNS is also known as the efferent division.

FALSE Afferent flows towards the CNS (Sensory). Efferent flows away from the CNS.

Describe cranial nerve I: The Olfactory Nerves

Arise from the olfactory receptor cells of nasal cavity Pass through the cribriform plate of the ethmoid bone Fibers synapse in the olfactory bulbs Pathway terminates in the primary olfactory cortex (temporal lobe) Purely sensory (olfactory) function Chemoreceptors

Describe cranial nerve II: The Optic Nerves

Arise from the retinas Pass through the optic canals, converge and partially cross over at the optic chiasma Optic tracts continue to the thalamus, where they synapse Optic radiation fibers run to the occipital (visual) cortex Purely sensory (visual) function

Describe the Lumbar Plexus

Arises from L1-L4 (mostly cauda equina) Innervates the thigh, abdominal wall, and psoas muscle Includes two important nerves: Femoral nerve—innervates quadriceps and skin of anterior thigh and medial surface of leg (lower) Obturator nerve—passes through obturator foramen to innervate adductor muscles (muscles & skin of medial thigh)

Describe the Sacral Plexus

Arises from L4-S4 Serves the buttock, lower limb, pelvic structures, and perineum Includes the Sciatic nerve: - Longest and thickest nerve of the body (about 1" wide) - Innervates the hamstring muscles, adductor magnus, and most muscles in the leg and foot (all of lower extremeties except for anterior thigh) - Composed of two nerves: tibial (posterior leg & sole of foot) and common fibular (knee & anterior leg, dorsum of foot)

B) the facial nerve (VII)

As a cook chops red onions he begins to tear up due to activation of the lacrimal gland. Which of the following nerves provided the stimulus? A) the olfactory nerve (I) B) the facial nerve (VII) C) the vagus nerve (X) D) the optic nerve (II)

Choose the FALSE statement about nerves

Axons make up a majority of the matter in a nerve

Choose the FALSE statement about nerves.

Axons make up a majority of the matter in a nerve

Choose the FALSE statement about nerves. Nerves can carry only sensory information, only motor information, or a mixture of sensory and motor information. Axons make up a majority of the matter in a nerve. Nerve axons are surrounded by a loose connective tissue layer called the endoneurium. Nerves consist of parallel bundles of myelinated and nonmyelinated axons.

Axons make up a majority of the matter in a nerve.

Identify the choroid.

B The choroid is the part of the vascular layer that lies posterior to the cornea's edge and under the sclera.

Identify the muscle that is controlled by the abducens nerve (CN VI).

B The lateral rectus muscle rotates the eye laterally, therefore "abducting" the field of vision. This is the only muscle innervated by the abducens nerve.

Name three ways receptors are classified

Based on: - Stimulus type - Location - Structural complexity

D) characterized by paralysis of facial muscles

Bell's palsy is ________. A) characterized by partial paralysis of diaphragm muscles B) characterized by loss of vision C) often caused by inflammation of the trigeminal nerve D) characterized by paralysis of facial muscles

C) Have bill grip an object with his thumb and index finger and try to pull the object away.

Bill is a mechanic that works with vibrating tools. He also exerts force on his wrists when twisting wrenches and screws. Bill has a tingling sensation in the lateral portion of his hand. The doctor suspects carpal tunnel syndrome. Which of the following test might the doctor try on her patient? A) Have bill flex and extend his arm against resistance. B) Check for reflex on the medial condyle of the humerus with a rubber mallet. C) Have bill grip an object with his thumb and index finger and try to pull the object away. D) Check for hyperextension at the knuckles of the little and ring finger.

Wallerian degeneration

Changes that occur in the axon distal to the site of axonal damage; the axon swells up and becomes irregular; eventually, the axon and the terminals are broken down into fragments that are phagocytosed by adjacent macrophages and Schwann cells.

Identify the layer that contains both a single-celled pigmented layer and a neural layer.

C The inner layer (retina) contains both the pigmented layer and the neural layer.

Contrast the functions of the PNS & CNS. Explain why damage to the PNS nerve fiber is often reversible whereas damage to CNS fibers is rarely repaired.

CNS: includes the brain and spinal cord. Motor responses and higher cognitive thinking. PNS: include all other neural tissue outside of the CNS. Sends sensory data to CNS and sends motor responses to neural tissue of PNS. If the PNS nerve fibers are damaged, chemical and physical process from macrophages and shcwann cells can repair them. Meanwhile, the CNS has oligondendrosites which produce growth inhibitor proteins that disable the repair of nerve fibers in CNS.

Compare and contrast a graded potential vs. an action potential.

Charts from chapter 11!!

D) injury to any single spinal nerve will be less damaging as there is less chance of total loss of innervation to any particular organ

Complicated interlacing of the ventral rami form networks called nerve plexus. The crisscrossing of the nerve fibers from the various spinal nerves is advantageous because ________. A) viruses that infect us by moving through PNS nerves are prevented entry to the CNS B) having several nerve fibers unite enhances motor function C) having several nerve fibers unite enhances sensory function D) injury to any single spinal nerve will be less damaging as there is less chance of total loss of innervation to any particular organ

Ralph sustained a leg injury in a bowling accident and had to use crutches. Unfortunately, he never took the time to learn how to use them properly. After two weeks of use, he noticed his fingers were becoming numb. Then he noticed his arms were getting weaker and had a tingling sensation. What could be his problem?

Compression of the radial nerve (in the region of the armpit) may cause temporary cessation of nervous transmission, often called "Saturday night paralysis."

Precommand level

Controls the outputs of the cortex and regulates motor activity.

Marcus, a football quarterback, suffered torn menisci in his right knee joint when tackled from the side. The same injury crushed his common fibular nerve against the head of the fibula. What locomotor problems did Marcus have after this?

Damage to his common fibular nerve would result in problems dorsiflexing his right foot and his knee joint would be unstable (more rocking of the femur from side to side on the tibia).

What is a benefit of a nerve plexus?

Damage to one singe branch of a plexus does not necessarily disrupt all motor information sent to a region

What is a benefit of a nerve plexus?

Damage to one single branch of a plexus does not necessarily disrupt all motor information sent to a region.

As Luke fell off a ladder, he grabbed a tree branch with his right hand, but unfortunately lost his grip and fell heavily to the ground. Days later, Luke complained that his upper limb was numb. What was damaged in his fall?

Damage to the brachial plexus occurred when he suddenly stopped his fall by grabbing the branch.

A) they can help pinpoint the location of spinal injury

Dermatome maps are useful to clinicians because ________. A) they can help pinpoint the location of spinal injury B) they show doctors how to avoid striking spinal nerves during surgery C) they show the routes of motor nerves D) they outline the location of the numerous nerve plexus

(1.) Describe the formation and composition of a spinal nerve. (2.) Name the branches of a spinal nerve (other than the rami communicantes) and indicate their distribution.

Dermatomes are related to the sensory innervation regions of the spinal nerves. The spinal nerves correlate with the segmented body plan, as do the muscles (at least embryologically). 1. Spinal nerves form from dorsal and ventral roots that unite distal to the dorsal root ganglion. Spinal nerves are mixed nerves that contain both sensory and motor fibers. 2. The ventral rami, with the exception of those in the thorax that form the intercostal nerves, contribute to large plexuses that supply the anterior and posterior body trunk and limbs. The dorsal rami supply the muscles and skin of the back (posterior trunk).

True

Dorsal and ventral rami are similar in that they both contain sensory and motor fibers.

What do dorsal and ventral roots combine to create?

Dorsal and ventral roots unite to form spinal nerves (1-2 cm long before branching into rami), which then emerge from the vertebral column via the intervertebral foramina

Which of the receptor types contributes to the sense of touch by responding to light pressure?

E) The tactile (Merkel) disc shown in E would be located in the base of the epidermis and would detect light touch.

Which of these receptor types functions as an exteroceptor?

E) All of the structures illustrated are sensitive to stimuli arising outside the body.

List and describe three types of connective tissue enclosing a nerve

Endoneurium—loose connective tissue that encloses single axons and their myelin sheaths Perineurium—coarse connective tissue that bundles fibers into fascicles Epineurium—tough fibrous sheath around a nerve (enclosing fascicles + blood vessels, lymph)

List and describe classifications of sensory receptors based on location

Exteroceptors: respond to stimuli arising outside the body; receptors in the skin for touch, pressure, pain, and temperature (general sensation), as well as most special sense organs (taste, smell, sight, hearing, equilibrium) Interoceptors (visceroceptors): respond to stimuli arising in internal viscera and blood vessels (sensitive to chemical changes, tissue stretch, and temperature changes) Proprioceptors: respond to stretch in skeletal muscles, tendons, joints, ligaments, and connective tissue coverings of bones and muscles, informing the brain of one's movements (and location in space)

Sensory stimuli that activate receptors generate action potentials that are sent into the CNS

FALSE Sensory stimuli that activate receptors generate graded potentials in the receptor cell. These graded potentials ultimately pass the message on to a sensory neuron. If the graded potential is strong enough at the axon hillock it triggers an action potential that continues on to the CNS.

David, an aspiring baseball player, was struck on the left side of his face with a fastball pitch. He was not wearing a safety helmet. His zygomatic arch was crushed, as well as parts of the temporal bone. Following the accident and reconstructive surgery, he noted that his left lower eyelid was still drooping and the corner of his mouth sagged. What nerve damage did he sustain?

Facial nerve damage on his left side

CNS nerve fibers regenerate because of the actions of Schwann cells.

False

CNS nerve fibers regenerate because of the actions of Schwann cells. True False

False

Stretch reflexes can cause reciprocal inhibition because the sensory neuron synapses on the alpha motor neurons of both agonists and antagonists.

False

T or F? A dermatome represets the motor innervation of muslces in that area

False

T or F? Sensory stimuli that activate receptors generate action potentials that are sent into the CNS

False

T or F? Stretch reflexes can cause reciprocal inhibition because the sensory neuron synapses on the alpha motor neurons of both agonists and antagonists

False

Sensory stimuli that activate receptors generate action potentials that are sent into the CNS. True False

False Sensory stimuli that activate receptors generate graded potentials that are sent into the CNS.

Stretch reflexes can cause reciprocal inhibition because the sensory neuron synapses on the alpha motor neurons of both agonists and antagonists. True False

False The stretch reflex activates agonists directly. However, reciprocal inhibition of the antagonist happens through an interneuron.

C) Lamellar corpuscles

Feeling a gentle caress on your arm would likely involve all of the following except ________. A) Meissner's corpuscles B) tactile discs C) Lamellar corpuscles D) hair follicle receptors

Describe cranial nerve XII: The Hypoglossal Nerves

Fibers from the medulla exit the skull via the hypoglossal canal Innervate extrinsic and intrinsic muscles of the tongue that contribute to swallowing and speech

Describe cranial nerve IX: The Glossopharyngeal Nerves

Fibers from the medulla leave the skull via the jugular foramen and run to the throat MOTOR functions: innervate part of the tongue and pharynx for swallowing, and provide parasympathetic fibers to the parotid salivary glands SENSORY functions: fibers conduct taste and general sensory impulses from the pharynx and posterior tongue, and impulses from carotid chemoreceptors and baroreceptors

Which layer of the eye is indicated by letter A?

Fibrous The fibrous layer includes the tough sclera (the "white" of the eye) as well as the transparent cornea.

Compare and contrast flexor and crossed-extensor reflexes.

Flexor reflexes are protective ipsilateral and polysynaptic reflexes that are designed to pull a part of the body away from a painful stimulus. Crossed-extensor reflexes consist of an ipsilateral withdrawal reflex and a contralateral extensor reflex that usually aids in maintaining balance.

Describe the Brachial Plexus

Formed by ventral rami of C5-C8 and T1 (and often C4 and T2) It gives rise to the nerves that innervate the upper limb (arms) Major branches of this plexus: Roots—five ventral rami (C5-T1) Trunks—upper, middle, and lower Divisions—anterior and posterior Cords—lateral, medial, and posterior Branches - individual nerves VERY complicated! Know category names, not specifics "Really Tired? Drink Coffee (Buckets)!" OR "Robert Taylor Drinks Cold Beer"

Describe cranial nerve XI: The Accessory Nerves

Formed from ventral rootlets from the C1-C5 region of the spinal cord (not the brain) Rootlets pass into the cranium via each foramen magnum Accessory nerves exit the skull via the jugular foramina to innervate the trapezius and sternocleidomastoid muscles

Describe the Cervical Plexus

Formed mostly by ventral rami of C1-C4 (and a bit of C5) Innervates skin and muscles of the neck, ear, back of head, and some of the shoulders Includes Phrenic nerve: major motor and sensory nerve of the diaphragm (receives fibers from C3-C5) No need to know other names

Select the statement that is most correct.

Ganglia associated with afferent nerve fibers contain cell bodies of sensory neurons.

Short-lived, localized changes in membrane potential that can either be depolarization or hyper polarization are known as ____.

Graded potentials

List the twelve cranial nerves

I - Olfactory: smelling II - Optic: seeing III - Oculomotor: eye movement IV - Trochlear: eye movement V - Trigeminal: face, taste VI - Abducens: eye movement VII - Facial: facial sensations VIII - Vestibulocochlear: hearing & balance IX - Glossopharyngeal: tongue & throat X - Vagus: viscera XI - Accessory: neck & shoulders XII - Hypoglossal: below tongue

C) nociceptors

If someone spills very hot coffee (200°F) on their skin, they will likely perceive much pain. Which of the following receptor types is causing this sensation? A) mechanoreceptors B) thermoreceptors C) nociceptors D) chemoreceptors

B) a complete loss of voluntary movement

If the ventral root of a spinal nerve were cut, what would be the result in the tissue or region that nerve supplies? A) complete loss of sensation B) a complete loss of voluntary movement C) loss of neither sensation nor movement but only of autonomic control D) a complete loss of sensation and movement

B) extend

In a crossed-extensor reflex, if the right arm was grabbed it would flex and the left arm would ________. A) also flex B) extend C) abduct D) adduct

Explain why damage to the peripheral nerve fibers is often reversible, whereas damage to CNS fibers rarely is.

In the PNS, damaged fibers can be replaced or repaired by physical and chemical processes directed by macrophages and Schwann cells. In the CNS, oligodendrocytes do not aid fiber regeneration because they have growth-inhibiting proteins on their surface, allowing damaged fibers to collapse and die.

Name the muscle at D.

Inferior rectus The inferior rectus originates from the common tendinous ring at the back of the orbit and inserts into the inferior surface of the eye.

C) the phrenic nerve that serves the diaphragm receives its fibers from here

Injury to cervical vertebra C3-C4 is particularly problematic because ________. A) part of the brain stem is located here B) several ganglia are near this region that serve the heart C) the phrenic nerve that serves the diaphragm receives its fibers from here D) the greater auricular nerve that serve the parotid gland receive there fibers from here

True

Irritation of the phrenic nerve may cause diaphragm spasms called hiccups.

Feeling a gentle caress on your arm would likely involve all of the following except ________.

Lamellar corpuscles

True

Lamellar corpuscles can be exteroceptors, interoceptors, or proprioceptor.

Describe cranial nerve V: The Trigeminal Nerves

Largest cranial nerves; fibers extend from pons to face Three divisions Ophthalmic (V1) passes through the superior orbital fissure Maxillary (V2) passes through the foramen rotundum Mandibular (V3) passes through the foramen ovale Convey SENSORY impulses from various areas of the face (V1) and (V2), and supplies MOTOR fibers (V3) for mastication (chewing)

Describe Regeneration of Nerve Fibers

Mature neurons are amitotic (if cell body is damaged, regneration is unlikely) If the soma of a damaged PNS nerve is intact, axon will regenerate; this involves coordinated activity among: - Macrophages—remove debris - Schwann cells—form regeneration tube and secrete growth factors - Axons—regenerate damaged part CNS oligodendrocytes bear growth-inhibiting proteins that prevent CNS fiber regeneration

List and describe classifications of sensory receptors based on stimulus type

Mechanoreceptors—respond to touch, pressure, vibration, stretch, and itch Thermoreceptors—sensitive to changes in temperature Photoreceptors—respond to light energy (e.g., retina) Chemoreceptors—respond to chemicals (e.g., smell, taste, changes in blood chemistry) Nociceptors—sensitive to pain-causing stimuli (e.g. extreme heat or cold, excessive pressure, inflammatory chemicals) - can be trigged along with another type

In carpal tunnel syndrome, the __________ is compressed.

Median Nerve

The right and the left side of the cerebrum control opposite sites of the body because signals cross over at the ____.

Medulla Oblongata

The _____ coordinates complex autonomic reflexes.

Medulla oblongata

B) olfactory

Mixed cranial nerves containing both motor and sensory fibers include all except which of the following? A) oculomotor B) olfactory C) trigeminal D) facial

Which of the following is NOT correct concerning nerves? -Nerves are analogous to tracts in the CNS. -Nerves are collection of axons of either sensory or motor neurons but not both. -Nerves do not contain cell bodies. -Nerves are covered by an outer sheath called the epineurium.

Nerves are collection of axons of either sensory or motor neurons but not both. Nerves can be mixed collections of both sensory and motor axons.

Which of the following is NOT correct concerning nerves? Nerves are collection of axons of either sensory or motor neurons but not both. Nerves do not contain cell bodies. Nerves are analogous to tracts in the CNS. Nerves are covered by an outer sheath called the epineurium.

Nerves are collection of axons of either sensory or motor neurons but not both. Nerves can be mixed collections of both sensory and motor axons.

Select the correct definition.

Pattern recognition allows us to see a familiar face.

Define Perception

Perception: the conscious interpretation of those stimuli

The connective tissue sheath that surrounds a fascicle of nerve fibers in the

Perineurium

B) vestibulocochlear

Problems in balance may follow trauma to which nerve? A) abducens B) vestibulocochlear C) trigeminal D) accessory

Flexor

Produces a rapid withdrawal of the body part from a painful stimulus; ipsilateral.

Golgi tendon

Produces muscle relaxation and lengthening in response to tension; the contracting muscle relaxes as its antagonist is activated.

Tendon

Produces muscle relaxation and lengthening in response to tension; the contracting muscle relaxes as its antagonist is activated.

What are the three levels of neural integration in sensory systems?

Receptor level—the sensor receptors Circuit level—ascending pathways Perceptual level—neuronal circuits in the cerebral cortex

Reflex arch order

Receptor-sensory neuron-integrations center-motor neuron-effector

Olfactory

Receptors located in epithelium of the nasal cavity.

The steps of generating an action potential are _____.

Resting state; depolarization; repolarization; hyperpolarization.

Accessory

Rootlets arise on the spinal cord, enter the skull through the foramen magnum and exit the skull through the jugular foramen.

After axonal injury, regeneration in peripheral nerves is guided by ________.

Schwann cells

Which of the following sympathetic effects would be reversed by parasympathetic activity?

Secretion of sweat

List and describe types of sensory receptors with unencapsulated dendritic endings

See chart, p. 487 1. Thermoreceptors: - Cold receptors (10-40ºC); in superficial dermis - Heat receptors (32-48ºC); in deeper dermis 2. Nociceptors, responding to: - Pinching - Chemicals from damaged tissue - Temperatures outside the range of thermoreceptors - Capsaicin 3. Light touch receptors: - Tactile (Merkel) discs - Hair follicle receptors

C) Pattern recognition allows us to see a familiar face.

Select the correct definition. A) Magnitude estimation is the simplest level of sensation. B) Perceptual detection is the ability to detect how much stimulus is applied to the body. C) Pattern recognition allows us to see a familiar face. D) Spatial discrimination allows us to recognize textures.

B) Ganglia associated with afferent nerve fibers contain cell bodies of sensory neurons.

Select the statement that is most correct. A) Ganglia are collections of neuron cell bodies in the spinal cord that are associated with efferent fibers. B) Ganglia associated with afferent nerve fibers contain cell bodies of sensory neurons. C) The dorsal root ganglion is a motor-only structure. D) The cell bodies of afferent ganglia are located in the spinal cord.

A) mechanoreceptors that are also interoceptors

Some large arteries that are proximal to the heart are sensitive to the stretch of the blood vessels. This stretch indicates the blood's pressure. Which of the following pairs of classifications below best fit the receptor type that is being described above? A) mechanoreceptors that are also interoceptors B) thermoreceptors that are also interoceptors C) mechanoreceptors that are also exteroceptors D) thermoreceptors that are also exteroceptors

Describe sensory receptors

Specialized to respond to changes in their environment (stimuli) Activation results in graded potentials that trigger nerve impulses Sensation (awareness of stimulus) and perception (interpretation of the meaning of the stimulus) occur in the brain

C) sacral plexus

Spinal nerves exiting the cord from the level of L4 to S4 form the ________. A) lumbar plexus B) femoral plexus C) sacral plexus D) thoracic plexus

A) roots, trunks, divisions, and cords

Starting at the spinal cord, the subdivisions of the brachial plexus are (in order) ________. A) roots, trunks, divisions, and cords B) roots, divisions, cords, and trunks C) divisions, roots, trunks, and cords D) trunks, divisions, cords, and roots

A bit of dust blows into and touches the cornea of the eye. Which of the following is likely to happen?

Stimulation of the ophthalmic division of the trigeminal nerve (V) will cause blinking.

Brachial plexus

Striking the "funny bone" (ulnar nerve) may cause injury to a nerve of this plexus.

C) ulnar nerve

Striking the "funny bone" is actually stimulation of (or injury to) the ________. A) radial nerve B) sciatic nerve C) ulnar nerve D) median nerve

Describe the meaning and importance of sympathetic tone and parasympathetic tone?

Sympathetic tone determines constriction or dilation degree. This is in the vascular system under resting systems. Parasympathetic tone is important to determining the heart rate and GI function.

Which of the receptor types contributes to the sense of touch by responding to light pressure? free nerve endings of sensory neurons muscle spindles Lamellar (Pacinian) corpuscles Tactile (Merkel) discs

Tactile (Merkel) discs The tactile (Merkel) disc shown in E would be located in the base of the epidermis and would detect light touch.

A) mechanoreceptors

Tactile sensation is a combination of touch, pressure, stretch and vibration. Which of the following is most likely the receptor type that senses tactile stimulation? A) mechanoreceptors B) thermoreceptors C) nociceptors D) proprioceptors

Plantar

Tests both upper and lower motor pathways. The sole of the foot is stimulated with a dull instrument.

All processing at the circuit level going up to the perceptual level must synapse in the

Thalamus

Why are the cerebellum and basal nuclei called precommand areas?

The cerebellum is called a precommand area because it integrates inputs from all ascending tracts prior to these inputs reaching the cortical command centers. The basal nuclei play a role in inhibiting cortical areas of the brain, preventing response until this inhibition stops.

A nurse is asked about the cause of the excruciating pain of tic douloureux. How should the nurse answer?

The excruciating pain is caused by inflammation of the trigeminal nerve. Pressure on the trigeminal nerve root can turn normal stimuli, like tooth brushing, into painful stimuli.

D) musculocutaneous

The flexor muscles in the anterior arm (biceps brachii and brachialis) are innervated by what nerve? A) radial B) median C) ulnar D) musculocutaneous

What is the homeostatic value of flexor reflexes?

The flexor, or withdrawal, reflex is a protective mechanism to withdraw from a painful stimulus, leading to a loss of pain.

One of a group of rabbit hunters was accidentally sprayed with buckshot in both of his gluteal prominences. When his companions saw that he would survive, they laughed and joked about where he had been shot. They were horrified and ashamed a week later when it was announced that their friend would be permanently paralyzed and without sensation in both legs from the knee down, as well as on the back of his thighs. What had happened?

The region of motor and sensory loss follows the course of the sciatic nerves (and their divisions); they must have been severely damaged by the shooting accident.

Fumiko, a 19-year-old nursing student, had had a runny nose and sore throat for several days. Upon waking, her face felt "twisted." When she examined her face in the mirror, she noticed that the right side looked "droopy" and she was unable to move the facial muscles on that side. This made it difficult to eat or speak clearly. Which cranial nerve was affected and on which side? What is a common cause of this condition?

The right facial nerve was affected. This condition is called Bell's palsy and is commonly caused by a herpes simplex 1 viral infection.

True

The second cranial nerve forms a chiasma at the base of the brain for partial crossover of neural fibers.

Explain how a crossed-extensor reflex exemplifies both serial and parallel processing.

The sensory input of a crossed-extensor reflex illustrates parallel processing, an ipsilateral response to a stimulus. The serial processing phase consists of motor activity, the contralateral response that activates the extensor muscles on the opposite side of the body.

You are at a party at Emma's house. After you are blindfolded, an object (a key or a rabbit's foot) is placed in your hand. What spinal tracts carry the signals to the cortex that allow you to differentiate between these objects and what aspects of sensory perception are operating?

The specific ascending pathways of the fasciculus cuneatus carry discriminative touch information from the upper limbs to the cortex. You must use feature abstraction and possibly pattern recognition to identify a specific pattern feature such as the teeth of a key or the fur of a rabbit's foot.

B) exteroceptors

We can touch our finger to our nose while our eyes are closed in part because we can sense the position and movement of our joints as well as the length of stretch in our muscles. These sensations create awareness of our body's positioning. The following receptors are most likely responsible for this ability. A) nociceptors B) exteroceptors C) interoceptors D) proprioceptors

Which receptors adapt most slowly?

nociceptors

T or F? Phasic receptors adapt quickly to a stimulus. For that reason, they are good at detecting changes instead of constantly signaling the CNS

True

Describe cranial nerves

Twelve pairs of nerves associated with the brain Most are mixed in function; two pairs are purely sensory Each nerve is identified by a number (I through XII) and a name "On occasion, our trusty truck acts funny—very good vehicle anyhow" OR: "Oooh ooh ooh to touch and feel very good vagina aah hah!" For remembering sensory or motor function or both: "Some say marry money but my brother says big boobs mean more."

What do the ventral rami of T2 - T12 do?

Ventral rami of T2-T12 as intercostal nerves supply muscles of the ribs, anterolateral thorax, and abdominal wall

B) receptor, afferent neuron, integration center, efferent neuron, effector

Which of the following is the correct simple spinal reflex arc? A) effector, afferent neuron, integration center, efferent neuron, receptor B) receptor, afferent neuron, integration center, efferent neuron, effector C) effector, efferent neuron, integration center, afferent neuron, receptor D) receptor, efferent neuron, integration center, afferent neuron, effector

The medical chart of a 68-year-old man includes the following notes: "Slight tremor of right hand at rest; stony facial expression; difficulty in initiating movements." [a] Based on your present knowledge, what is the diagnosis? [b] What brain areas are most likely involved in this man's disorder, and what is the deficiency? [c] How is this condition currently treated?

[a] Most likely, the diagnosis would be that he probably has Parkinson's disease. [b] The brain areas that are most likely effected in his disorder are most likely the basal nuclei because the neurons of the substantia nigra are not being released properly. Recent knowledge says that the mitochondrial proteins that are abnormal or degrading can also play a part. He is deficient in dopamine which is causing his symptoms and disorder. [c] His condition is usually treated best by drug therapy. If patients no longer respond to this type of treatment, their best bet is to undergo deep brain stimulation via implanted electrodes because it shuts down any abnormal brain activity and can alleviate tremors. They could also undergo gene therapy which includes the insertion of genes into brain cells which causes them to inhibit certain neurotransmitters. There are many options for this type of disease although some can be pricy.

What differentiates an autonomic reflex from a somatic reflex?

a two-neuron motor pathway

The cranial nerve with a cervical origin (spinal cord) is the ________.

accessory

damage to this nerve would cause difficulty in speech and swallowing, but no effect on visceral organs

accessory

Classify the neuron at A.

afferent sensory neuron A is an afferent sensory neuron carrying information from the muscle to the CNS.

Which of the following adrenergic neurotransmitter receptors plays the major role in heart activity?

beta 1

What motor structure inhibits the motor cortex at rest? cerebellum brain stem motor nuclei basal nuclei precommand area

basal nuclei The basal nuclei inhibit motor centers at rest but release that inhibition to allow for motor activity.

survival center

brain stem

afferent sensory neuron

carry information from the muscle to the CNS

arbor vitae

cerebellum

motor command center

cerebellum

provides timing and patterns of skeletal muscle contraction for daily life

cerebellum

What parts of the brain ultimately plan and coordinate complex motor activities?

cerebellum and basal nuclei

executive suite

cerebrum

The primary nerve that controls breathing is found in which nerve plexus?

cervical

The phrenic nerve is a branch from the __________.

cervical plexus The phrenic nerve, which arises from the cervical plexus, supplies both motor and sensory fibers to the diaphragm, the main breathing muscle.

During close vision, what actions must the eye take to bring an object into focus?

contract the ciliary body

Sciatica has no direct affect on ______.

control of the adductor longus

A traumatic brain injury leading to permanent brain damage is a ____.

contusion

Which reflex has a contralateral component?

corssed-extensor

Which pair below is incorrect?

crainal Nerve IV; sensory

Which pair below is incorrect? cranial nerve III: pupillary constriction cranial nerve IV: sensory cranial nerve VI: eye movement cranial nerve III: oculomotor nerve optic nerve: sensory

cranial nerve IV: sensory The trochlear nerve is a motor nerve that innervates the superior oblique of the eye.

Which reflex has a contralateral component?

crossed-extensor

Which reflex is triggered when a stranger suddenly grasps your arm?

crossed-extensor

What is the specific function of the structure labeled F?

determination of muscle length The muscle spindle indicated by F functions as a proprioceptor that is responsive to changes in the length of the surrounding muscle.

The hyperalgesia that is common in phantom limb pain could be blocked if a new drug was developed that could prevent (without triggering any side-effects) the ______.

diffusion of calcium ions through NMDA receptors

The hyperalgesia that is common in phantom limb pain could be blocked if a new drug was developed that could prevent (without triggering any side-effects) the ______. diffusion of calcium ions through NMDA receptors release of enkephalins binding of enkephalins to NMDA receptors diffusion of enkephalins across synaptic clefts

diffusion of calcium ions through NMDA receptors Diffusion of calcium ions through these receptors cause a depolarizing event and activate and action potential in the neuron.

Tonic Receptors

do NOT exhibit the property of adaptation. ( Nociceptors and most proprioceptors are tonic receptors)

motor speech area

frontal

premotor area

frontal

seat of intelligence, abstract reasoning

frontal

somatic motor cortex

frontal

what is responsible for the overall integration of the autonomic nervous system (ANS)?

hypothalamus

Which of the following is a way in which the somatic and autonomic nervous systems are similar?

not similar

If "wrist drop" were to appear, there would also be an increased probability of ______.

inability to extend the forearm

Complicated interlacing of the ventral rami form networks called nerve plexus. The crisscrossing of the nerve fibers from the various spinal nerves is advantageous because ________.

injury to any single spinal nerve will be less damaging as there is less chance of total loss of innervation to any particular organ

taste area

insula

Which of the following is NOT required for a reflex arc?

interneurons In many cases, the interneurons are the integration point for reflexes, but they are not necessary.

The outflow of the ANS

is characterized by a two-neuron chain spanning from CNS to the effector organ.

Raynaud's disease ________.

is characterized by exaggerated vasoconstriction in the extremities

Regeneration within the CNS ________.

is prevented due to growth-inhibiting proteins of oligodendrocytes

the obturator and femoral nerves branch from this plexus

lumbar

Some large arteries that are proximal to the heart are sensitive to the stretch of the blood vessels. This stretch indicates the blood's pressure. Which of the following pairs of classifications below best fit the receptor type that is being described above?

mechanoreceptors that are also interoceptors

Spinal nerves are all classified as

mixed nerves

Spinal nerves are all classified as __________.

mixed nerves

When removing a hot sheet of cookies from the oven, you feel heat coming through the pot holder. What mechanism momentarily overrides your reflexive action to immediately drop the hot cookie sheet?

modulation of descending motor pathways by brainstem and cortex

When removing a hot sheet of cookies from the oven, you feel heat coming through the pot holder. What mechanism momentarily overrides your reflexive action to immediately drop the hot cookie sheet? modification of outgoing efferent signals by spinal cord gray matter interneurons temporary suppression of sensory input to the spinal reflex center intervention by the limbic system to inhibit the fear of being burned modulation of descending motor pathways by brainstem and cortex

modulation of descending motor pathways by brainstem and cortex

visual area

occipital

A fracture of the ethmoid bone could result in damage to which cranial nerve?

olfactory

Mixed cranial nerves containing both motor and sensory fibers include all except which of the following?

olfactory

A patient is suffering from the inability to distinguish various types of odors. This patient may have damage to which of the following?

olfactory nerve (I)

Which division of the nervous system has long preganglionic neurons?

parasympathetic

rest and digest

parasympathetic

A person picks up a heavy suitcase in order to estimate its weight and reflexively drops it. Which of the following receptors has initiated this reflex?

tendon organ

primary sensory cortex

parietal

In order for you to realize there has been a sensory change, it has to reach the ___________________ level of processing

perceptual

controls the outputs of the cortex and regulates motor activity

projection

includes cortical and brain stem motor areas

projection

intermediate relay for incoming and outgoing neurons

projection

contain autonomic nerve fibers

rami communicantes

The first level of neural integration in the somatosensory system is the __________ level.

receptor

Which of the following lists the hierarchy of motor control from lowest to highest level of control?

segmental level, projection level, precommand level

Somatic reflexes activate __________.

skeletal muscle

A herniated lumbar disc could interfere with ______.

skin sensations from the lateral thigh skin sensations from the medial thigh adduction of the thigh

Segmental refers to which level of motor control? brain stem spinal cord cerebellum basal nuclei

spinal cord

Which structure is involved in the segmental level of motor control?

spinal cord

The brachial plexus can be palpated at the lower lateral border of the sternocleidomastoid muscle. Injury to the brachial plexus could cause weakness or paralysis to all of the following EXCEPT the

sternocleidomastoid

Transduction refers to conversion of ________.

stimulus energy into energy of a graded potential

The knee-jerk reflex is an example of a __________.

stretch reflex

The knee-jerk reflex is an example of a __________. flexor reflex superficial reflex tendon reflex stretch reflex

stretch reflex

Which reflex is important for maintaining muscle tone?

stretch reflex

fight or flight

sympathetic

Which of the following releases the neurotransmitter norepinephrine?

terminus of a sympathetic postganglionic neuron

A doctor asks her patient to follow the motion of her finger as she moves it up and down, left and right. Which of the following cranial nerves is not being tested?

the vestibulocochlear nerve (VIII)

__________ do NOT exhibit the property of adaptation.

tonic receptors

Which of the following characteristics is representative of receptor-level processing, NOT perceptual-level processing?

transduciton

Which of the following characteristics is representative of receptor-level processing, NOT perceptual-level processing? magnitude transduction detection spatial discrimination

transduction Transduction is converting a stimulus into a nervous signal; this happens at the receptor level.

A knee-jerk reflex that is unusually strong may be caused by ______.

transmission of excitatory signals from the brain to the neurons that form the femoral nerve

helps to regulate blood pressure and digestion

vagus

Which cranial nerve innervates most of the visceral organs? accessory vagus abducens hypoglossal

vagus The vagus nerve has many targets in the thoracic and abdominal cavities and innervates many of the visceral organs.

Which cranial nerve innervates most of the visceral organs?

vagus nerve The vagus nerve has many targets in the thoracic and abdominal cavities and innervates many of the visceral organs.

Which cranial nerve transmits information about audition? optic vestibulocochlear vagus abducens

vestibulocochlear The vestibular branch of the vestibulocochlear nerve transmits afferent impulses for the sense of equilibrium. The cochlear branch transmits afferent impulses for the sense of hearing.


Conjuntos de estudio relacionados

Exam 2 Multiple Choice Semester 1

View Set

Cost Accounting 1-2 - Variable and Full Costing

View Set

The PCV system controls which exhaust emission(s)?

View Set

we done did it OB is over <3333 :)

View Set

Chapter 01: Introduction to Nursing

View Set

The Call of the Wild-Chapter 4 Questions

View Set